Solution space of linear homogeneous PDE forms a vector space?

Click For Summary
The discussion centers on the claim that the solution space of a linear homogeneous partial differential equation (PDE) forms a vector space. The proof provided demonstrates that if two solutions exist, their linear combinations also solve the PDE, satisfying the necessary conditions for a vector space. However, there is confusion regarding the need to prove additional vector space properties beyond the two demonstrated. It is suggested that some properties may be implicitly assumed based on the context of functions as a subset of a larger vector space. Ultimately, the focus is on clarifying the sufficiency of the proof and the assumptions made about the function space.
kingwinner
Messages
1,266
Reaction score
0

Homework Statement


Claim:
The solution space of a linear homogeneous PDE Lu=0 (where L is a linear operator) forms a "vector space".

Proof:
Assume Lu=0 and Lv=0 (i.e. have two solutions)
(i) By linearity, L(u+v)=Lu+Lv=0
(ii) By linearity, L(au)=a(Lu)=(a)(0)=0
=> any linear combination of the solutions of a linear homoegenous PDE solves the PDE
=> it forms a vector space


Homework Equations


N/A

3. The Attempt at a Solution and comments
Now, I don't understand why ONLY by proving (i) and (ii) alone would lead us to conclude that it is a vector space. There are like TEN properties that we have to prove before we can say that it is a vector sapce, am I not right?
Are there any theorem or alternative definition that they have been using?

Thanks!
 
Last edited:
Physics news on Phys.org
maybe consider the other axioms and test them, some may have been considered more bovious, but always worth checking
 
Other things are satisfied by the definition of functions, for example u+v=v+u is obvious and not a property of being a solution of a PDE. It may be implicitly assumed that you're talking about the subset of the vector space of all functions from U to V (where U and V are Rm and Rn such that you'd be looking for solutions with that domain and codomain). In that case you only have to prove the two subspace properties
 
Question: A clock's minute hand has length 4 and its hour hand has length 3. What is the distance between the tips at the moment when it is increasing most rapidly?(Putnam Exam Question) Answer: Making assumption that both the hands moves at constant angular velocities, the answer is ## \sqrt{7} .## But don't you think this assumption is somewhat doubtful and wrong?

Similar threads

  • · Replies 1 ·
Replies
1
Views
2K
Replies
2
Views
1K
Replies
15
Views
2K
  • · Replies 2 ·
Replies
2
Views
2K
  • · Replies 23 ·
Replies
23
Views
3K
  • · Replies 9 ·
Replies
9
Views
4K
Replies
9
Views
2K
  • · Replies 8 ·
Replies
8
Views
2K
  • · Replies 36 ·
2
Replies
36
Views
5K
Replies
2
Views
2K